Search found 51 matches


Also, any two perpendicular lines will have the product of their slopes as -1. So you can also take the question as asking 'is the product of their slopes -1 ?' Now, we can find out the slope of a line if we know two points it passes through. The question itself gives us one point for each line. Sta...

by jitsy

Tue Jul 16, 2013 12:47 pm
Forum: Data Sufficiency
Topic: it isnt tough but i still didnt get it
Replies: 2
Views: 967

Line K passes through point (1,1) and M passes through (1,-1). Draw those two points and you will realise that one is in 1st quadrant and other in 4th. This is what the question stem tells you. And the question is are the two lines perpendicular. Statement 1: Tells us that lines intersect at point 1...

by jitsy

Tue Jul 16, 2013 12:41 pm
Forum: Data Sufficiency
Topic: it isnt tough but i still didnt get it
Replies: 2
Views: 967

Great story woojet! a BIG congratulations! You removed the 'un' from 'unremarkable'. Reminds me of that scene from Pursuit of Happyness where Will Smith says to his son - "Don't ever let someone tell you, you can't do something. Not even me. You got a dream, you got to protect it. People can’...


I know you addressed the question to Brent - but would just like to tell you that I have come across questions like these (votes and candidates) in the past and what the test makers are almost always testing here is if the person falls for the trap - which is assuming that there are only 2 candidate...

by jitsy

Tue Jul 16, 2013 8:48 am
Forum: Data Sufficiency
Topic: which is the right answer ?
Replies: 7
Views: 2722

This involves conditional probability. The probability of A conditioned on B, denoted P(A|B), is equal to P(AB)/P(B). The division provides that the probabilities of all outcomes within B will sum to 1. Conditioning restricts the sample space to those outcomes which are in the set being conditioned ...

by jitsy

Tue Jul 16, 2013 7:37 am
Forum: Problem Solving
Topic: Probablility PS
Replies: 5
Views: 2081

In an election how many votes are cast. 1. the winning candidate gets 54% of the total votes 2. the winning candidate wins by 800 As per the solution asnwer is E But my solution is C Please suggest me which is the right one. Thanks in advance. Regards, Uva. Hi Uva@90 I hope you did not read the sec...

by jitsy

Tue Jul 16, 2013 6:23 am
Forum: Data Sufficiency
Topic: which is the right answer ?
Replies: 7
Views: 2722

Thank you.

Yes, the question did come with an explanation which more or less matched with your previous explanation. Is there a way you know I can let Princeton know about this so that this can be rectified in the future editions of the book? Thank you.

by jitsy

Wed Jul 10, 2013 1:26 am
Forum: GMAT Math
Topic: Princeton Review - Wrong answer and explanation?
Replies: 11
Views: 6973

Thanks a lot! Really appreciate your detailed explanation. However, if you consider √2 and -√2 to be X and Y respectively, both statement 1) and statement 2) are satisfied [√2*(-√2) = -2 which is an integer and √2 + (-√2) = 0 which is also an integer]. And of course, √2 and -√2 are n...

by jitsy

Tue Jul 09, 2013 12:42 pm
Forum: GMAT Math
Topic: Princeton Review - Wrong answer and explanation?
Replies: 11
Views: 6973

Krishp84 (and JennySpalek this should help you as well), I guess you calculated the LCM for both numerators and denominators. The HCF of 3 and 10 (the highest number common in them) is 1 [3=1*3 and 10=1*2*5]. So the answer to the question JennySpalek asked should infact be 4/1 or just 4 (and not 4/3...

by jitsy

Tue Jul 09, 2013 4:26 am
Forum: GMAT Math
Topic: Papgust's GMAT MATH FLASHCARDS directory
Replies: 146
Views: 98497

Princeton Review - Wrong answer and explanation?

Hi everyone, I encountered the question below in Drill 11 of the Quant section of 'Cracking the New GMAT 2013' by Princeton Review. Are X and Y integers? (1) the product XY is an integer (2) X + Y is an integer The book states 'C' as the answer - I think the answer could be E. Can someone who has lo...

by jitsy

Mon Jul 08, 2013 8:23 pm
Forum: GMAT Math
Topic: Princeton Review - Wrong answer and explanation?
Replies: 11
Views: 6973

Hi nk_81, The answer would have been B had the question stem asked "The answer to which of the following questions would be most relevant to evaluating the conclusion?" That is, if were supposed to find out if the octane levels are really less, then we would have asked "ok, if its rea...

by jitsy

Thu Jan 06, 2011 11:20 am
Forum: Critical Reasoning
Topic: Tricky CR!!
Replies: 5
Views: 1673

Thanks Jim!! Appreciate your response.

by jitsy

Mon Dec 27, 2010 11:14 am
Forum: Sentence Correction
Topic: 1000 SC ques# 134
Replies: 9
Views: 3680

Hi Gaap, You didn't state exactly when you're taking the GMAT. If you have sufficient time, I suggest taking a mock to see where you are placed. You might already be at a 540. Plus, with a bit of brushing up of concepts and grammar, you might get there. If after that, you think you still need to joi...

by jitsy

Sat Dec 25, 2010 12:40 pm
Forum: GMAT Strategy
Topic: Suggest me a prep course.
Replies: 2
Views: 837

Thanks Rezinka. Totally agree with you and no doubts there. Would like to use your second example to understand the problem 2 actions in past : First - ice sheets HAD existed Second - the studies 'led' him to propose in 1837 So, A is correct. It indicates that the studies led him to propose in 1837 ...

by jitsy

Sat Dec 25, 2010 4:47 am
Forum: Sentence Correction
Topic: SC problems
Replies: 6
Views: 1106

Thanks a lot Ravish & Rezinka. 1. Thanks, the use of whether for 2 or more alternatives is what I didnt know. Would have gone with the correct ans otherwise. 2. Thanks Ravish for the explanation. But my question of 'like' vs 'as' remains unanswered. In general, what Ive been noticing is that, as...

by jitsy

Sat Dec 25, 2010 3:40 am
Forum: Sentence Correction
Topic: SC problems
Replies: 6
Views: 1106